LSAT and Law School Admissions Forum

Get expert LSAT preparation and law school admissions advice from PowerScore Test Preparation.

 fg6118
  • Posts: 26
  • Joined: Apr 18, 2016
|
#24065
"What we need is an answer that links the sufficient part of our conclusion (important and well written) to our premises in the correct way so that we can follow the arrows in the reasoning chain and end up with our necessary term (promoted) in the conclusion.

Answer (A) does this by linking: (Important -> published) in the correct sufficient/necessary order. The fact that this answer only mentions "important" and not "well written" doesn't matter because it still justifies the argument. In other words, if "important" guarantees that it gets published, the fact that it is also well written doesn't matter."

I'm trying to understand the fact that in a justify question, when we're proving the conclusion 100%, it's ok to choose an answer that has 1 of the 2 terms of the conclusion's compound sufficient condition linked to a premise's sufficient condition to make the logical chain work and reach the necessary condition of the conclusion.

This not having to strictly have both elements of the sufficient condition in play makes me wonder how else it could happen. What if that first premise of this stimulus had a compound sufficient condition, like "if Skiff's book is published this year AND IS MADE AVAILABLE AS AN E-BOOK, Professor Nguyen vows she will urge the dean to promote Skiff." Would answer A still work?

Thanks!
 Nikki Siclunov
PowerScore Staff
  • PowerScore Staff
  • Posts: 1362
  • Joined: Aug 02, 2011
|
#24075
Hi Fara,

Thanks for your question! You'll find an answer to your question in my first response to this above:
As you state, answer choice (A) immediately justifies the conclusion, because it provides the assumption which, when added to the stimulus’ premises, fully justifies the author’s conclusion. It is not necessary that answer choice (A) identify both of the qualities mentioned in the conclusion. Remember - this is a Justify question: the correct answer choice, if true, must prove the conclusion. Thus, if the book's proclaimed importance is sufficient to establish that the book will be published, the conclusion is automatically justified regardless of whether Skiff's book is well-written.
However, given the hypothetical argument you propose, answer choice would not be correct. This is because answer choice (A) would only serve to justify one of the two sufficient conditions in the compound proposition, which would not be enough to trigger the chain and justify the necessary condition (Skiff promoted).

Does that help?

Thanks!
 fg6118
  • Posts: 26
  • Joined: Apr 18, 2016
|
#24101
That makes sense. Thank you!
 avengingangel
  • Posts: 275
  • Joined: Jun 14, 2016
|
#30932
I know this has been explained/diagrammed above, but I'm just not getting it... Before getting to the question stem, I'm diagramming the argument as such:

Premise - Published :arrow: Prof urges to promote "Skiff" (first sentence)
Premise - Prof urges to promote Skiff :arrow: gets promoted (end of 2nd sentence)
Conclusion - important + well written :arrow: gets promoted (beginning of 2nd sentence)

Combine the first two premises to get:
Premise - Published :arrow: Prof urges to promote Skiff :arrow: gets promoted
Conclusion - important + well written :arrow: gets promoted

Cool. Then I make the conditional inference in the first diagram to get:
Premise - Published :arrow: gets promoted
Conclusion - important + well written :arrow: gets promoted

hmm, interesting, I have "gets promoted" as the nec. condition in both my stimulus & premise.. duly noted. * reads stimulus * Nice, it's a justify question. So I know I need to somehow "connect" this whole "important + well written" thing with the "published" thing. I'm thinking the "important" thing should be the necessary condition, since it's the suf. condition in the conclusion, therefore leaving "published" to be the sufficient condition. * reads answer choices * Hm, A almost got me, but clearly it's Mistaken Reversal! Ah... D, yes, you look good. * circles D * * feels good * * moves on to #22 *

Can you tell me what I did wrong here in my reasoning ?! I feel like it should be the reverse of A!

Thanks :-)
 Adam Tyson
PowerScore Staff
  • PowerScore Staff
  • Posts: 5153
  • Joined: Apr 14, 2011
|
#31307
You were great on your diagrams, Angel, until the very end. The premises are how we prove that Skiff will be promoted, so we need to make those premises necessary. Answer A does that - it makes "published" necessary, and then the rest of your chain follows and everything points to Skiff getting promoted, the conclusion that we wanted to justify.

D is the one that gets things backwards, because D does nothing to make the chain in our premises necessary. We want to prove that Skiff gets published so that we can then connect the dots all the way to promotion. D gives us that if he gets published, in addition to all the other stuff happening in the chain, the book must also be important and well written. But how do we get to proving that it will, in fact, be published, and thus Skiff gets a promotion? Try adding answer A to your diagram, and also try adding answer D, and you'll see that A completes the chain while D simply makes a new branch.

Here's a formula I give my students for dealing with tricky conditional justify questions. This make take a few read-throughs to absorb:

1) If you have a conditional premise and a conditional conclusion that you want to justify, and the premise and conclusion have matching necessary conditions, you justify the conclusion by making the sufficient condition in the conclusion sufficient for the sufficient condition in the premise. That looks like this, where P is a premise, C is a conclusion, and J is a correct Justify answer:
P: A -> B
C: C -> B
J: C -> A

2) If you have a conditional premise and a conditional conclusion that you want to justify, and the premise and conclusion have matching sufficient conditions, you justify the conclusion by making the necessary condition in the premise sufficient for the necessary condition in the conclusion . That looks like this:

P: D -> E
C: D -> F
J: E -> F

That formula, once memorized (and hopefully internalized so that it becomes more intuitive and less mechanical), should make questions like this one much easier and clearer, and you will have no trouble telling the right answer from the mistaken reversal or some other answer choice.

Give that a look and see what happens when you apply it here. Good luck!
 avengingangel
  • Posts: 275
  • Joined: Jun 14, 2016
|
#32508
Ah, that was so wonderful, thank you, Adam!

To be sure, #1 would apply to this question, yes ??

I will definitely work on memorizing & internalizing these! They make sense. Thanks again !!
 Adam Tyson
PowerScore Staff
  • PowerScore Staff
  • Posts: 5153
  • Joined: Apr 14, 2011
|
#32570
You got it, angel! Well done. You're welcome!
 akanshalsat
  • Posts: 104
  • Joined: Dec 20, 2017
|
#49961
i think one of the staff, in an explanation for A said:

Answer (A) does this by linking: (Important -> published) in the correct sufficient/necessary order. The fact that this answer only mentions "important" and not "well written" doesn't matter because it still justifies the argument. In other words, if "important" guarantees that it gets published, the fact that it is also well written doesn't matter.

but if there is an "AND" involved (even if its in the sufficient condition), doesn't it mean you MUST do both in order to get the necessary condition to follow through? isn't that what is taught in the And vs. or part? if it said important OR well written ---> promotion, then I think A makes sense bc its satisfying the important part but the AND makes it important to satisfy both parts of the sufficient condition?

Super confused if I completely studied/understood the sufficient v. necessary and the and v. or wrong

I get why D is wrong though, bc it flips the sufficient and necessary - and i got this question right --> i just don't get how its the best answer
 Robert Carroll
PowerScore Staff
  • PowerScore Staff
  • Posts: 1787
  • Joined: Dec 06, 2013
|
#62759
akansha,

Answer choice (A) says that if it's as important as Skiff claimed, it will be published. But if it's also well-written, will that hurt anything? Of course not. Answer choice (A) is establishing that importance alone guarantees publication. Adding additional factors won't affect that relation - it's still important, so it's still published (so says the answer). So if it's important and well-written, it will be published. If it's important and written in Greek, it will be published. Answer choice (A) makes all the other factors irrelevant by linking importance and publication directly.

Keep in mind the relative strengths of the following conditionals:

A
+ :arrow: C
B

versus:

A :arrow: C

The latter conditional is stronger, because "If A then C" is true regardless of whether B is true or false. It basically makes B irrelevant - you can have it if you want, but you don't need it.

The same relation exists between answer choice (A) and the conditional you'd expect to connect to the stimulus. The stimulus was relying on "If the book is well-written and important, it will be published." A statement like "If it's important, it will be published" makes irrelevant how well-written the book was - importance implies publication directly. That's good for this Justify question.

If, on the other hand, we have a stimulus that relies on an unstated premise like "If this class is attended by at least 50 students, then it will be offered again next year," an answer that says "If this class is attended by at least 50 students and audited by at least 10, then it will be offered again next year" isn't good enough. This latter conditional tells me it will be offered again if BOTH of two conditions are met, but tells me nothing about the single condition, whereas the stimulus was relying on an occurrence stemming from the single condition. This wouldn't work. It's the exact opposite of what we have here, so this failure isn't a failure for our situation.

Robert Carroll
User avatar
 cornflakes
  • Posts: 48
  • Joined: Feb 19, 2021
|
#86053
Hi Powerscore,

I am still tripping over the same question that a few students have brought up about the AND idea not being perfectly adhered to in this question. My conclusion is that I must have an inherently flawed understanding of what compound sufficient conditions mean.

I interpret the conclusion for this argument as follows: "If Skiff's book is as important (as he claims) AND as well written as he claims, he will be promoted"

My understanding here is that the conclusion is the entire conditional statement: In essence, what we are trying to justify is the conditional statement itself, not just the outcome that he will get promoted. I can definitely see how answer A links "important as he says" to "promoted" by answer A, which would definitely justify the outcome of the conditional. Yet, I am continuing to fail to see how it justifies the application/validity of the conditional conclusion as a whole.

I have read the rationale of "we don't care what happens with the "well written" segment now because we know that "important" singlehandedly justifies "promoted", which in turn automatically justifies "important" AND "well written" lead to "promoted." In essence, if you know A causes C, then you can combine A,B,D,E,F,G,H,I,J,K,L,etc. and still know that this combo yields "C." So far, this makes sense. But then what if say E is sufficient to C not occurring - then we have a bit of a conundrum. We have a sufficient that leads to the necessary and a sufficient that says the necessary does not occur. Since we have no information about E (the added variables that aren't mentioned), we definitely can't assume this, so I suppose we must assume that they don't act in this way.

The only way I can square my confusion here is an inherent truth about sufficient conditions altogether that I haven't grasped - if something is sufficient, then it does not matter what else it is paired with, so long as that condition exists, the necessary will always follow (due to the inherent truth of something being "sufficient").

Is this a correct way of thinking about this problem? How do you deal with the possibility that conditions with unknown outcomes (they could be sufficient for negating the necessary condition) could be paired with sufficient conditions that bring about the necessary condition?

Thanks

Get the most out of your LSAT Prep Plus subscription.

Analyze and track your performance with our Testing and Analytics Package.